Why are prop blades not shaped like household fan blades?What are the advantages of more than 4 propeller blades?At what point is a turbine powered ducted propeller considered a turbofan?What’s this vortex generator like thing below the wing?How does the Angle of attack vary from the root to the tip of a propeller for a fixed pitch prop?Why don't helicopter blades look like other propellers?Why are aircraft external lights round, not tear-drop shaped?Why don't aeroplanes use propellers with more blades like the intake on a fanjet?Why do propeller blades not have winglets?What would happen if the blades of a variable pitch prop did not all have the same pitch?Why are RC rotor blades different from helicopter blades?Which are some planes shaped like an inverted wing?

I'm from Mumbai, India. I want to travel to Europe as a tourist but my salary is low ( €190/month)

Is policy routing bad?

Chess evaluation function

Longest word worth at most a million

threading a compression clamp

Really bizarre dystopian children’s film with hundreds of young boys forced to play piano

Paying to leave without notice in at-will employment state

The Extended Participial Phrase

Jazz Chords: How can a Bbmaj9 be explained in the key of A major?

Why do Muslim refugees seek asylum in Europe and not in rich countries in the Middle East?

How to delete file in folder except name files in list.txt?

Download, upload, downstream, upstream

Why use Fourier series instead of Taylor?

Why was the DC-9-80 so successful despite being obsolete almost from birth?

What was meant by the protest sign "Bundestag nach Berlin"?

What DC should I use for someone trying to survive indefinitely solely with an alchemy jug as their only source of food and water? (survival campaign)

Equipment replacement problem

Invalid time zone 'UTC'

Commutative algebras with modules of small complexity

My mysterious "ruins" wander around and change on their own, what'd be a rational way for them to do that?

Sylow theorem and tetrahedron

Where is a warlock's soul?

Equality operator does not get defined for a custom spaceship operator implementation in C++20

Short story about two entangled quantum physicists



Why are prop blades not shaped like household fan blades?


What are the advantages of more than 4 propeller blades?At what point is a turbine powered ducted propeller considered a turbofan?What’s this vortex generator like thing below the wing?How does the Angle of attack vary from the root to the tip of a propeller for a fixed pitch prop?Why don't helicopter blades look like other propellers?Why are aircraft external lights round, not tear-drop shaped?Why don't aeroplanes use propellers with more blades like the intake on a fanjet?Why do propeller blades not have winglets?What would happen if the blades of a variable pitch prop did not all have the same pitch?Why are RC rotor blades different from helicopter blades?Which are some planes shaped like an inverted wing?






.everyoneloves__top-leaderboard:empty,.everyoneloves__mid-leaderboard:empty,.everyoneloves__bot-mid-leaderboard:empty
margin-bottom:0;









34














$begingroup$


My two projects right now are learning to fly and 3D modeling a replacement blade for antique fan, and a question occurred to me:



Why are airplane propeller blades not shaped like household fan blades?



They're both designed to push the same fluid with some degree of efficiency, but household fan blades look a lot more like ship screws/boat propeller blades. Is it a weight issue? A visibility issue? A drag issue, and if so, how?



EDIT - This is not a duplicate of this question because the proposed answers a question about angle of attack and pitch, which are aviation jargon words that may mean nothing to novices. Pitch and angle of attack also have little to do with the front-view shape of a household fan or prop blade, which is likely the most obvious difference between the two types of blades in the eyes of someone without an aeronautical engineering degree.










share|improve this question












$endgroup$














  • $begingroup$
    Possible duplicate of How does the Angle of attack vary from the root to the tip of a propeller for a fixed pitch prop?
    $endgroup$
    – Ryan Mortensen
    Aug 1 at 21:42






  • 4




    $begingroup$
    @RyanMortensen That seems like a completely different question to me. That question is asking about the definition of the term "angle of attack" and the mathematics around it; this question is asking about the motivation behind a design decision.
    $endgroup$
    – Terran Swett
    Aug 1 at 23:31










  • $begingroup$
    I'm not going to entertain a debate, this isn't the place. Ask on meta. Based on feedback I've seen from far more experienced users in the past, the main determining factor seems to be based on the resultant answers of a question, not the actual question itself.
    $endgroup$
    – Ryan Mortensen
    Aug 2 at 1:35







  • 1




    $begingroup$
    @RyanMortensen, I agree with Tanner. The question you proposed does not answer my question, and even if it did, is not phrased in a way a relative newcomer would ever have asked it. I worded my question as I did because it is exactly the question I wanted to ask.
    $endgroup$
    – schadjo
    Aug 2 at 12:38






  • 1




    $begingroup$
    You might want to post this on engineering as well, because the answers here don't get at the underlying issues in any coherent way. There are certainly differences in the fluid dynamics. These are characterized by differences in Reynolds number, thrust coefficient, and power coefficient between the two. And fans are often ducted to some degree. But the real difference is the cost of achieving acceptable performance.
    $endgroup$
    – Phil Sweet
    Aug 2 at 21:10


















34














$begingroup$


My two projects right now are learning to fly and 3D modeling a replacement blade for antique fan, and a question occurred to me:



Why are airplane propeller blades not shaped like household fan blades?



They're both designed to push the same fluid with some degree of efficiency, but household fan blades look a lot more like ship screws/boat propeller blades. Is it a weight issue? A visibility issue? A drag issue, and if so, how?



EDIT - This is not a duplicate of this question because the proposed answers a question about angle of attack and pitch, which are aviation jargon words that may mean nothing to novices. Pitch and angle of attack also have little to do with the front-view shape of a household fan or prop blade, which is likely the most obvious difference between the two types of blades in the eyes of someone without an aeronautical engineering degree.










share|improve this question












$endgroup$














  • $begingroup$
    Possible duplicate of How does the Angle of attack vary from the root to the tip of a propeller for a fixed pitch prop?
    $endgroup$
    – Ryan Mortensen
    Aug 1 at 21:42






  • 4




    $begingroup$
    @RyanMortensen That seems like a completely different question to me. That question is asking about the definition of the term "angle of attack" and the mathematics around it; this question is asking about the motivation behind a design decision.
    $endgroup$
    – Terran Swett
    Aug 1 at 23:31










  • $begingroup$
    I'm not going to entertain a debate, this isn't the place. Ask on meta. Based on feedback I've seen from far more experienced users in the past, the main determining factor seems to be based on the resultant answers of a question, not the actual question itself.
    $endgroup$
    – Ryan Mortensen
    Aug 2 at 1:35







  • 1




    $begingroup$
    @RyanMortensen, I agree with Tanner. The question you proposed does not answer my question, and even if it did, is not phrased in a way a relative newcomer would ever have asked it. I worded my question as I did because it is exactly the question I wanted to ask.
    $endgroup$
    – schadjo
    Aug 2 at 12:38






  • 1




    $begingroup$
    You might want to post this on engineering as well, because the answers here don't get at the underlying issues in any coherent way. There are certainly differences in the fluid dynamics. These are characterized by differences in Reynolds number, thrust coefficient, and power coefficient between the two. And fans are often ducted to some degree. But the real difference is the cost of achieving acceptable performance.
    $endgroup$
    – Phil Sweet
    Aug 2 at 21:10














34












34








34


2



$begingroup$


My two projects right now are learning to fly and 3D modeling a replacement blade for antique fan, and a question occurred to me:



Why are airplane propeller blades not shaped like household fan blades?



They're both designed to push the same fluid with some degree of efficiency, but household fan blades look a lot more like ship screws/boat propeller blades. Is it a weight issue? A visibility issue? A drag issue, and if so, how?



EDIT - This is not a duplicate of this question because the proposed answers a question about angle of attack and pitch, which are aviation jargon words that may mean nothing to novices. Pitch and angle of attack also have little to do with the front-view shape of a household fan or prop blade, which is likely the most obvious difference between the two types of blades in the eyes of someone without an aeronautical engineering degree.










share|improve this question












$endgroup$




My two projects right now are learning to fly and 3D modeling a replacement blade for antique fan, and a question occurred to me:



Why are airplane propeller blades not shaped like household fan blades?



They're both designed to push the same fluid with some degree of efficiency, but household fan blades look a lot more like ship screws/boat propeller blades. Is it a weight issue? A visibility issue? A drag issue, and if so, how?



EDIT - This is not a duplicate of this question because the proposed answers a question about angle of attack and pitch, which are aviation jargon words that may mean nothing to novices. Pitch and angle of attack also have little to do with the front-view shape of a household fan or prop blade, which is likely the most obvious difference between the two types of blades in the eyes of someone without an aeronautical engineering degree.







aerodynamics propeller






share|improve this question
















share|improve this question













share|improve this question




share|improve this question








edited Aug 2 at 12:40







schadjo

















asked Jul 31 at 18:45









schadjoschadjo

2932 silver badges8 bronze badges




2932 silver badges8 bronze badges














  • $begingroup$
    Possible duplicate of How does the Angle of attack vary from the root to the tip of a propeller for a fixed pitch prop?
    $endgroup$
    – Ryan Mortensen
    Aug 1 at 21:42






  • 4




    $begingroup$
    @RyanMortensen That seems like a completely different question to me. That question is asking about the definition of the term "angle of attack" and the mathematics around it; this question is asking about the motivation behind a design decision.
    $endgroup$
    – Terran Swett
    Aug 1 at 23:31










  • $begingroup$
    I'm not going to entertain a debate, this isn't the place. Ask on meta. Based on feedback I've seen from far more experienced users in the past, the main determining factor seems to be based on the resultant answers of a question, not the actual question itself.
    $endgroup$
    – Ryan Mortensen
    Aug 2 at 1:35







  • 1




    $begingroup$
    @RyanMortensen, I agree with Tanner. The question you proposed does not answer my question, and even if it did, is not phrased in a way a relative newcomer would ever have asked it. I worded my question as I did because it is exactly the question I wanted to ask.
    $endgroup$
    – schadjo
    Aug 2 at 12:38






  • 1




    $begingroup$
    You might want to post this on engineering as well, because the answers here don't get at the underlying issues in any coherent way. There are certainly differences in the fluid dynamics. These are characterized by differences in Reynolds number, thrust coefficient, and power coefficient between the two. And fans are often ducted to some degree. But the real difference is the cost of achieving acceptable performance.
    $endgroup$
    – Phil Sweet
    Aug 2 at 21:10

















  • $begingroup$
    Possible duplicate of How does the Angle of attack vary from the root to the tip of a propeller for a fixed pitch prop?
    $endgroup$
    – Ryan Mortensen
    Aug 1 at 21:42






  • 4




    $begingroup$
    @RyanMortensen That seems like a completely different question to me. That question is asking about the definition of the term "angle of attack" and the mathematics around it; this question is asking about the motivation behind a design decision.
    $endgroup$
    – Terran Swett
    Aug 1 at 23:31










  • $begingroup$
    I'm not going to entertain a debate, this isn't the place. Ask on meta. Based on feedback I've seen from far more experienced users in the past, the main determining factor seems to be based on the resultant answers of a question, not the actual question itself.
    $endgroup$
    – Ryan Mortensen
    Aug 2 at 1:35







  • 1




    $begingroup$
    @RyanMortensen, I agree with Tanner. The question you proposed does not answer my question, and even if it did, is not phrased in a way a relative newcomer would ever have asked it. I worded my question as I did because it is exactly the question I wanted to ask.
    $endgroup$
    – schadjo
    Aug 2 at 12:38






  • 1




    $begingroup$
    You might want to post this on engineering as well, because the answers here don't get at the underlying issues in any coherent way. There are certainly differences in the fluid dynamics. These are characterized by differences in Reynolds number, thrust coefficient, and power coefficient between the two. And fans are often ducted to some degree. But the real difference is the cost of achieving acceptable performance.
    $endgroup$
    – Phil Sweet
    Aug 2 at 21:10
















$begingroup$
Possible duplicate of How does the Angle of attack vary from the root to the tip of a propeller for a fixed pitch prop?
$endgroup$
– Ryan Mortensen
Aug 1 at 21:42




$begingroup$
Possible duplicate of How does the Angle of attack vary from the root to the tip of a propeller for a fixed pitch prop?
$endgroup$
– Ryan Mortensen
Aug 1 at 21:42




4




4




$begingroup$
@RyanMortensen That seems like a completely different question to me. That question is asking about the definition of the term "angle of attack" and the mathematics around it; this question is asking about the motivation behind a design decision.
$endgroup$
– Terran Swett
Aug 1 at 23:31




$begingroup$
@RyanMortensen That seems like a completely different question to me. That question is asking about the definition of the term "angle of attack" and the mathematics around it; this question is asking about the motivation behind a design decision.
$endgroup$
– Terran Swett
Aug 1 at 23:31












$begingroup$
I'm not going to entertain a debate, this isn't the place. Ask on meta. Based on feedback I've seen from far more experienced users in the past, the main determining factor seems to be based on the resultant answers of a question, not the actual question itself.
$endgroup$
– Ryan Mortensen
Aug 2 at 1:35





$begingroup$
I'm not going to entertain a debate, this isn't the place. Ask on meta. Based on feedback I've seen from far more experienced users in the past, the main determining factor seems to be based on the resultant answers of a question, not the actual question itself.
$endgroup$
– Ryan Mortensen
Aug 2 at 1:35





1




1




$begingroup$
@RyanMortensen, I agree with Tanner. The question you proposed does not answer my question, and even if it did, is not phrased in a way a relative newcomer would ever have asked it. I worded my question as I did because it is exactly the question I wanted to ask.
$endgroup$
– schadjo
Aug 2 at 12:38




$begingroup$
@RyanMortensen, I agree with Tanner. The question you proposed does not answer my question, and even if it did, is not phrased in a way a relative newcomer would ever have asked it. I worded my question as I did because it is exactly the question I wanted to ask.
$endgroup$
– schadjo
Aug 2 at 12:38




1




1




$begingroup$
You might want to post this on engineering as well, because the answers here don't get at the underlying issues in any coherent way. There are certainly differences in the fluid dynamics. These are characterized by differences in Reynolds number, thrust coefficient, and power coefficient between the two. And fans are often ducted to some degree. But the real difference is the cost of achieving acceptable performance.
$endgroup$
– Phil Sweet
Aug 2 at 21:10





$begingroup$
You might want to post this on engineering as well, because the answers here don't get at the underlying issues in any coherent way. There are certainly differences in the fluid dynamics. These are characterized by differences in Reynolds number, thrust coefficient, and power coefficient between the two. And fans are often ducted to some degree. But the real difference is the cost of achieving acceptable performance.
$endgroup$
– Phil Sweet
Aug 2 at 21:10











7 Answers
7






active

oldest

votes


















54
















$begingroup$

1) Airspeed, 2) Forward motion, 3) Size constraints. Just to begin with.



Household fan blades are extremely slow, so they need more chord to push a meaningful amount of air. Aircraft propellers approach the speed of sound at their tips, and low drag is critical. All things equal, more span and less chord is more efficient. Reducing the airspeed for props has diminishing returns, because aircraft themselves move forward through the air, so a very large propeller adds drag to the aircraft however slow it is.



In household fans, size constraints are more critical than efficiency. A high-span, low-chord (narrow) blade would be more efficient in a household fan as well - indeed, you can find household fans with such blades. But they have to either spin faster for the same amount of airflow, which adds noise, or be larger in diameter. Ceiling fans, which can be larger, have longer narrower blades than desk or floor-standing fans.



Ship propellers move in an extremely dense and viscous medium, which changes things even more. The drag of moving through water is extremely high and proportional to V³ power-wise. The thrust force they produce is at most proportional to V², and can be lost to cavitation at high velocity. So their velocity has to be kept as low as possible. The drag cost of adding more chord is also relatively small in water.



On large ships, propellers are already as large as they can be made without sticking out of the water at low draft (on merchant vessels) or reducing the number of shafts that can fit (on combatants). This allows them to spin slower and lose less power to drag.






share|improve this answer












$endgroup$










  • 6




    $begingroup$
    Fan blades spinning as fast as propellers would also be a safety hazard, and also probably have extra trouble with vibrations. It's just a bad idea all around :D
    $endgroup$
    – Luaan
    Aug 1 at 7:10






  • 1




    $begingroup$
    The key reason for this is that efficiency loss mainly occurs at the tip of a blade (where high pressure air can leak around to the low pressure side). Fat blades have more tip than narrow blades, so they're less efficient.
    $endgroup$
    – Robin Bennett
    Aug 1 at 11:24






  • 3




    $begingroup$
    Submarines, which have no "sticking out of the water" problem, can have incredibly large props.
    $endgroup$
    – Carl Witthoft
    Aug 1 at 12:41










  • $begingroup$
    @CarlWitthoft submarines have to work to a degree on the surface. And a screw that sticks out of the water is one someone can take a picture of... if the boat comes out of the water in drydock the screw gets shrouded. It might be more accurate to say that submarines have very cramped hulls.
    $endgroup$
    – Chris Stratton
    Aug 1 at 14:57







  • 4




    $begingroup$
    @Luaan Some household fan blades actually do spin at similar RPMs to light aircraft propellers. They just tend to be rather small fans and usually have a shroud around them to prevent any danger. And they're also not nearly as long, so the blade tips aren't moving nearly as fast for the same RPMs. Also, not nearly as much mass per blade. Computer cooling fans are a common example of this. It's not unusual for them to be in the 1,500 - 2,000 RPM range, which is around the same as the PA-28s I've flown.
    $endgroup$
    – reirab
    Aug 1 at 16:48



















22
















$begingroup$

From our local warehouse



Some household fans are shaped like airplane propellers, the ones that need to move a lot of air at the highest speed. For a given motor, they have the highest efficiency, but the tip effect makes them noisy. They are best suited for industrial applications.



For inside the family home there are other considerations:



  • Silence. Best if we don’t hear the fan run at all.

  • Low air speed. We only want to feel the cooling effect of moving air, not have our hair blown out of shape.

  • Purchase cost. This is the one we directly see. The usage costs are hidden in the monthly electricity bill.

So for that purpose, a slow moving tip with many broad blades is best.






share|improve this answer












$endgroup$














  • $begingroup$
    Let's try helicopter vs ceiling fan: helicopter - symmetrical airfoil. Ceiling fan - flat. Note long thin fan blades will help reduce interblade turbulence issues for a high speed industrial fan, but optimal airmoving shape that produces the LOWEST "lift" and LEAST DRAG would likely be a thin plate.
    $endgroup$
    – Robert DiGiovanni
    Jul 31 at 22:12







  • 6




    $begingroup$
    @RobertDiGiovanni ceiling fans tend towards flat blades because of their lower production cost and because they are often intended to operate in reverse with some efficiency, not because an airfoil would be bad. I have owned several standing fans with airfoil shaped blades, as well.
    $endgroup$
    – AEhere supports Monica
    Jul 31 at 22:24






  • 2




    $begingroup$
    "usage costs are hidden in the monthly electricity bill." Maybe about 0.5-2$ USD per month?
    $endgroup$
    – chux - Reinstate Monica
    Aug 1 at 7:06






  • 1




    $begingroup$
    @chux $2 per month adds up to the purchase price of a cheap desktop fan in a year. Of course you can pay 10 times as much for a fan that does exactly the same thing, if you want to :)
    $endgroup$
    – alephzero
    Aug 1 at 11:23


















8
















$begingroup$

I think it should be obvious that a fan blade is basically a wing travelling in a circle. So the question is what makes a wing a good wing?



In theory and in practice (wind tunnel measurements or fuel consumption measurements of real planes) a long skinny wing is significantly more efficient than a short wide wing. The most efficient wing for a given airfoil is actually one with infinite wingspan. But since there are only a finite number of electrons in the universe it is not possible to build this perfect wing. Instead engineers try to make wings/propellers/fan blades as long and as skinny as practical given the limitations of materials.



You will even see this effect in household fan blades: in situations where span is not limited, such as ceiling fans, the fan blades tend to be long and skinny instead of short and wide like ship propellers. So why short and wide? Space restriction.



When the fan must fit on a tabletop, you cannot have 1 meter long blades. But as soon as this restriction is lifted you will see designers shift to long skinny blades like what you can find on industrial floor standing or wall mounted fans.



Granted, there are floor standing fans with the exact same blades as tabletop fans but this has more to do with reusing parts and economies of scale than aerodynamics.



Now, you can in theory also have an efficient short skinny fan on a tabletop fan: just look at drone/RC airplane propellers - they're often way shorter than the fan blades of tabletop fans. But you will need to move the blades really fast to move a given amount of air. This is very noisy (have you ever seen a drone fly?). So the second consideration is noise. To reduce noise you move the blades slowly. A slow blade doesn't move much air so you increase the chord to make them wide. This results in a very inefficient fan but efficiency is not your main concern: you are designing a non-moving machine sitting on a table drawing power from a socket in the wall. Noise reduction and size matters more.






share|improve this answer










$endgroup$










  • 4




    $begingroup$
    +1 As an example, I used to pass through the VLC airport in the past, and it has massive fans on the ceiling, with long and thin blades not unlike the wings on a glider, and with a similar airfoil too. The fact that building maintenance often had them rotating backwards can be explained via Hanlon's razor. EDIT: see this question aviation.stackexchange.com/questions/61833/…
    $endgroup$
    – AEhere supports Monica
    Aug 1 at 12:56



















2
















$begingroup$

Learning to flying and modelling a fan blade, both spin in the air but the propeller tries to move the plane, and the fan tries to move the air.



How does this factor into design? Propellers stand above and apart from fans in that they generate lift not only by deflecting air (bottom lift), but also from their motion through the air (top lift). This is best explained by viewing the lift curve vs AOA of an airfoil from 0 to 45 degrees. Lift will increase up to stall, then decrease, then increase again up to 45 degrees. This means you have to move LESS air to get the same lift with an airfoil up to stall.



The fan really is opposite, unless you want it to pull it self around the room. The fan blade is designed to move air, period. Sensible design would make it compact, and thin flat blades with a wider chord would be fine, as one just looking for a cooling breeze from their product. Keeping the original style of fan on the "antique" may make it more valuable.






share|improve this answer












$endgroup$










  • 2




    $begingroup$
    You have a good point, but it doesn't need to involve the dubious concepts of 'top' and 'bottom' lift. Simply put, propellers are optimised to create lift (thrust) while disturbing the air as little as possible. This favours a large diameter in the first place, to move more air with the lowest possible speed. A fan is almost the opposite: it needs a certain diameter and move the air at a decent speed. (A cooling fan, that is; requirements may be different for a ventilation fan).
    $endgroup$
    – Zeus
    Aug 1 at 1:24










  • $begingroup$
    "A cooling fan, that is; requirements may be different for a ventilation fan" which may be different from a jet fan! Look at the shape of those airfoils!
    $endgroup$
    – Robert DiGiovanni
    Aug 1 at 6:10






  • 1




    $begingroup$
    @Zeus Tanner Swett published an airfoil Clift vs AOA graph that shows the "top lift" contribution at (lower drag) lower AOA as a lift maxima, which drops to a minima at around 30 degrees AOA, before rising to a maxima (with loads of drag) at 45 degrees. This is the Rosetta Stone for many applications. However, your point that the moving wing "disturbs as little air as possible" is an excellent way of describing it (hence less drag for a given lift!). Thanks.
    $endgroup$
    – Robert DiGiovanni
    Aug 1 at 6:19










  • $begingroup$
    @RobertDiGiovanni Friendly nitpick: consider using the singular form of minimum and maximum when appropriate.
    $endgroup$
    – AEhere supports Monica
    Aug 1 at 7:37


















2
















$begingroup$

Price is the main reason. Ceiling fans just stir the air around and flat paddles are the cheapest way to go.



More expensive fan blades can have some aerofoil shape and even winglets at the tips but this is mainly for show since these blades don't have twist.






share|improve this answer










$endgroup$














  • $begingroup$
    But wouldn't a more wing-shaped, narrower blade use less material and be less expensive?
    $endgroup$
    – schadjo
    Aug 1 at 15:08



















1
















$begingroup$

Most of the other answers are already correct. Just one more aspect: Blade chord length.



With something that spins as fast as an airplane propeller, the blades need to be as light as possible, to minimize centrifugal loads. So it's better to give them a short profile chord length and high area loading (how much force they produce per surface area of blade), which means small chord length and turning the flow comparatively sharply, which requires a large pressure difference between the pressure and suction side of the fan blades. For a desk fan, any old plastic will be strong enough, so those blades can work with a very low surface loading (long profile chord length) and turn the flow slowly, with a small pressure difference, which is quieter.



The same applies for ship propellers: Since water is so dense, the structural loading on those blades is pretty high. If you tried to get the same thrust from a propeller with short chords (at equal radius), the blades would have trouble carrying the bending loads, and the sharper turning would also lead to cavitation [1] quicker.



[1]: cavitation happens when the pressure on the suction side of an underwater propeller falls so low that water evaporates. This effectively separates the flow, which reduces suction, so the freshly-evaporated water condenses again, water rushes back to where the steam bubble was... That can break a propeller very quickly.






share|improve this answer










$endgroup$






















    1
















    $begingroup$

    Some rare designs do look more like a fan blade. The Antonov AN-70 is a good example of this, sporting huge fan blades on its D-27 propfan. The scimitar props do make for a rather efficient design, but they are extremely noisy which limits their usability around many commercial airports.



    enter image description here



    Image credit : By Tangopaso - Own work, CC BY-SA 3.0




    Some additional discussion can be found in this question :



    What are the advantages of more than 4 propeller blades?






    share|improve this answer










    $endgroup$














    • $begingroup$
      Those props are wild.
      $endgroup$
      – schadjo
      Aug 13 at 12:11












    Your Answer








    StackExchange.ready(function()
    var channelOptions =
    tags: "".split(" "),
    id: "528"
    ;
    initTagRenderer("".split(" "), "".split(" "), channelOptions);

    StackExchange.using("externalEditor", function()
    // Have to fire editor after snippets, if snippets enabled
    if (StackExchange.settings.snippets.snippetsEnabled)
    StackExchange.using("snippets", function()
    createEditor();
    );

    else
    createEditor();

    );

    function createEditor()
    StackExchange.prepareEditor(
    heartbeatType: 'answer',
    autoActivateHeartbeat: false,
    convertImagesToLinks: false,
    noModals: true,
    showLowRepImageUploadWarning: true,
    reputationToPostImages: null,
    bindNavPrevention: true,
    postfix: "",
    imageUploader:
    brandingHtml: "Powered by u003ca class="icon-imgur-white" href="https://imgur.com/"u003eu003c/au003e",
    contentPolicyHtml: "User contributions licensed under u003ca href="https://creativecommons.org/licenses/by-sa/4.0/"u003ecc by-sa 4.0 with attribution requiredu003c/au003e u003ca href="https://stackoverflow.com/legal/content-policy"u003e(content policy)u003c/au003e",
    allowUrls: true
    ,
    noCode: true, onDemand: true,
    discardSelector: ".discard-answer"
    ,immediatelyShowMarkdownHelp:true
    );



    );














    draft saved

    draft discarded
















    StackExchange.ready(
    function ()
    StackExchange.openid.initPostLogin('.new-post-login', 'https%3a%2f%2faviation.stackexchange.com%2fquestions%2f67177%2fwhy-are-prop-blades-not-shaped-like-household-fan-blades%23new-answer', 'question_page');

    );

    Post as a guest















    Required, but never shown


























    7 Answers
    7






    active

    oldest

    votes








    7 Answers
    7






    active

    oldest

    votes









    active

    oldest

    votes






    active

    oldest

    votes









    54
















    $begingroup$

    1) Airspeed, 2) Forward motion, 3) Size constraints. Just to begin with.



    Household fan blades are extremely slow, so they need more chord to push a meaningful amount of air. Aircraft propellers approach the speed of sound at their tips, and low drag is critical. All things equal, more span and less chord is more efficient. Reducing the airspeed for props has diminishing returns, because aircraft themselves move forward through the air, so a very large propeller adds drag to the aircraft however slow it is.



    In household fans, size constraints are more critical than efficiency. A high-span, low-chord (narrow) blade would be more efficient in a household fan as well - indeed, you can find household fans with such blades. But they have to either spin faster for the same amount of airflow, which adds noise, or be larger in diameter. Ceiling fans, which can be larger, have longer narrower blades than desk or floor-standing fans.



    Ship propellers move in an extremely dense and viscous medium, which changes things even more. The drag of moving through water is extremely high and proportional to V³ power-wise. The thrust force they produce is at most proportional to V², and can be lost to cavitation at high velocity. So their velocity has to be kept as low as possible. The drag cost of adding more chord is also relatively small in water.



    On large ships, propellers are already as large as they can be made without sticking out of the water at low draft (on merchant vessels) or reducing the number of shafts that can fit (on combatants). This allows them to spin slower and lose less power to drag.






    share|improve this answer












    $endgroup$










    • 6




      $begingroup$
      Fan blades spinning as fast as propellers would also be a safety hazard, and also probably have extra trouble with vibrations. It's just a bad idea all around :D
      $endgroup$
      – Luaan
      Aug 1 at 7:10






    • 1




      $begingroup$
      The key reason for this is that efficiency loss mainly occurs at the tip of a blade (where high pressure air can leak around to the low pressure side). Fat blades have more tip than narrow blades, so they're less efficient.
      $endgroup$
      – Robin Bennett
      Aug 1 at 11:24






    • 3




      $begingroup$
      Submarines, which have no "sticking out of the water" problem, can have incredibly large props.
      $endgroup$
      – Carl Witthoft
      Aug 1 at 12:41










    • $begingroup$
      @CarlWitthoft submarines have to work to a degree on the surface. And a screw that sticks out of the water is one someone can take a picture of... if the boat comes out of the water in drydock the screw gets shrouded. It might be more accurate to say that submarines have very cramped hulls.
      $endgroup$
      – Chris Stratton
      Aug 1 at 14:57







    • 4




      $begingroup$
      @Luaan Some household fan blades actually do spin at similar RPMs to light aircraft propellers. They just tend to be rather small fans and usually have a shroud around them to prevent any danger. And they're also not nearly as long, so the blade tips aren't moving nearly as fast for the same RPMs. Also, not nearly as much mass per blade. Computer cooling fans are a common example of this. It's not unusual for them to be in the 1,500 - 2,000 RPM range, which is around the same as the PA-28s I've flown.
      $endgroup$
      – reirab
      Aug 1 at 16:48
















    54
















    $begingroup$

    1) Airspeed, 2) Forward motion, 3) Size constraints. Just to begin with.



    Household fan blades are extremely slow, so they need more chord to push a meaningful amount of air. Aircraft propellers approach the speed of sound at their tips, and low drag is critical. All things equal, more span and less chord is more efficient. Reducing the airspeed for props has diminishing returns, because aircraft themselves move forward through the air, so a very large propeller adds drag to the aircraft however slow it is.



    In household fans, size constraints are more critical than efficiency. A high-span, low-chord (narrow) blade would be more efficient in a household fan as well - indeed, you can find household fans with such blades. But they have to either spin faster for the same amount of airflow, which adds noise, or be larger in diameter. Ceiling fans, which can be larger, have longer narrower blades than desk or floor-standing fans.



    Ship propellers move in an extremely dense and viscous medium, which changes things even more. The drag of moving through water is extremely high and proportional to V³ power-wise. The thrust force they produce is at most proportional to V², and can be lost to cavitation at high velocity. So their velocity has to be kept as low as possible. The drag cost of adding more chord is also relatively small in water.



    On large ships, propellers are already as large as they can be made without sticking out of the water at low draft (on merchant vessels) or reducing the number of shafts that can fit (on combatants). This allows them to spin slower and lose less power to drag.






    share|improve this answer












    $endgroup$










    • 6




      $begingroup$
      Fan blades spinning as fast as propellers would also be a safety hazard, and also probably have extra trouble with vibrations. It's just a bad idea all around :D
      $endgroup$
      – Luaan
      Aug 1 at 7:10






    • 1




      $begingroup$
      The key reason for this is that efficiency loss mainly occurs at the tip of a blade (where high pressure air can leak around to the low pressure side). Fat blades have more tip than narrow blades, so they're less efficient.
      $endgroup$
      – Robin Bennett
      Aug 1 at 11:24






    • 3




      $begingroup$
      Submarines, which have no "sticking out of the water" problem, can have incredibly large props.
      $endgroup$
      – Carl Witthoft
      Aug 1 at 12:41










    • $begingroup$
      @CarlWitthoft submarines have to work to a degree on the surface. And a screw that sticks out of the water is one someone can take a picture of... if the boat comes out of the water in drydock the screw gets shrouded. It might be more accurate to say that submarines have very cramped hulls.
      $endgroup$
      – Chris Stratton
      Aug 1 at 14:57







    • 4




      $begingroup$
      @Luaan Some household fan blades actually do spin at similar RPMs to light aircraft propellers. They just tend to be rather small fans and usually have a shroud around them to prevent any danger. And they're also not nearly as long, so the blade tips aren't moving nearly as fast for the same RPMs. Also, not nearly as much mass per blade. Computer cooling fans are a common example of this. It's not unusual for them to be in the 1,500 - 2,000 RPM range, which is around the same as the PA-28s I've flown.
      $endgroup$
      – reirab
      Aug 1 at 16:48














    54














    54










    54







    $begingroup$

    1) Airspeed, 2) Forward motion, 3) Size constraints. Just to begin with.



    Household fan blades are extremely slow, so they need more chord to push a meaningful amount of air. Aircraft propellers approach the speed of sound at their tips, and low drag is critical. All things equal, more span and less chord is more efficient. Reducing the airspeed for props has diminishing returns, because aircraft themselves move forward through the air, so a very large propeller adds drag to the aircraft however slow it is.



    In household fans, size constraints are more critical than efficiency. A high-span, low-chord (narrow) blade would be more efficient in a household fan as well - indeed, you can find household fans with such blades. But they have to either spin faster for the same amount of airflow, which adds noise, or be larger in diameter. Ceiling fans, which can be larger, have longer narrower blades than desk or floor-standing fans.



    Ship propellers move in an extremely dense and viscous medium, which changes things even more. The drag of moving through water is extremely high and proportional to V³ power-wise. The thrust force they produce is at most proportional to V², and can be lost to cavitation at high velocity. So their velocity has to be kept as low as possible. The drag cost of adding more chord is also relatively small in water.



    On large ships, propellers are already as large as they can be made without sticking out of the water at low draft (on merchant vessels) or reducing the number of shafts that can fit (on combatants). This allows them to spin slower and lose less power to drag.






    share|improve this answer












    $endgroup$



    1) Airspeed, 2) Forward motion, 3) Size constraints. Just to begin with.



    Household fan blades are extremely slow, so they need more chord to push a meaningful amount of air. Aircraft propellers approach the speed of sound at their tips, and low drag is critical. All things equal, more span and less chord is more efficient. Reducing the airspeed for props has diminishing returns, because aircraft themselves move forward through the air, so a very large propeller adds drag to the aircraft however slow it is.



    In household fans, size constraints are more critical than efficiency. A high-span, low-chord (narrow) blade would be more efficient in a household fan as well - indeed, you can find household fans with such blades. But they have to either spin faster for the same amount of airflow, which adds noise, or be larger in diameter. Ceiling fans, which can be larger, have longer narrower blades than desk or floor-standing fans.



    Ship propellers move in an extremely dense and viscous medium, which changes things even more. The drag of moving through water is extremely high and proportional to V³ power-wise. The thrust force they produce is at most proportional to V², and can be lost to cavitation at high velocity. So their velocity has to be kept as low as possible. The drag cost of adding more chord is also relatively small in water.



    On large ships, propellers are already as large as they can be made without sticking out of the water at low draft (on merchant vessels) or reducing the number of shafts that can fit (on combatants). This allows them to spin slower and lose less power to drag.







    share|improve this answer















    share|improve this answer




    share|improve this answer








    edited Aug 1 at 21:11









    Toby Speight

    8855 silver badges15 bronze badges




    8855 silver badges15 bronze badges










    answered Jul 31 at 19:01









    TheracTherac

    11.1k1 gold badge33 silver badges52 bronze badges




    11.1k1 gold badge33 silver badges52 bronze badges










    • 6




      $begingroup$
      Fan blades spinning as fast as propellers would also be a safety hazard, and also probably have extra trouble with vibrations. It's just a bad idea all around :D
      $endgroup$
      – Luaan
      Aug 1 at 7:10






    • 1




      $begingroup$
      The key reason for this is that efficiency loss mainly occurs at the tip of a blade (where high pressure air can leak around to the low pressure side). Fat blades have more tip than narrow blades, so they're less efficient.
      $endgroup$
      – Robin Bennett
      Aug 1 at 11:24






    • 3




      $begingroup$
      Submarines, which have no "sticking out of the water" problem, can have incredibly large props.
      $endgroup$
      – Carl Witthoft
      Aug 1 at 12:41










    • $begingroup$
      @CarlWitthoft submarines have to work to a degree on the surface. And a screw that sticks out of the water is one someone can take a picture of... if the boat comes out of the water in drydock the screw gets shrouded. It might be more accurate to say that submarines have very cramped hulls.
      $endgroup$
      – Chris Stratton
      Aug 1 at 14:57







    • 4




      $begingroup$
      @Luaan Some household fan blades actually do spin at similar RPMs to light aircraft propellers. They just tend to be rather small fans and usually have a shroud around them to prevent any danger. And they're also not nearly as long, so the blade tips aren't moving nearly as fast for the same RPMs. Also, not nearly as much mass per blade. Computer cooling fans are a common example of this. It's not unusual for them to be in the 1,500 - 2,000 RPM range, which is around the same as the PA-28s I've flown.
      $endgroup$
      – reirab
      Aug 1 at 16:48













    • 6




      $begingroup$
      Fan blades spinning as fast as propellers would also be a safety hazard, and also probably have extra trouble with vibrations. It's just a bad idea all around :D
      $endgroup$
      – Luaan
      Aug 1 at 7:10






    • 1




      $begingroup$
      The key reason for this is that efficiency loss mainly occurs at the tip of a blade (where high pressure air can leak around to the low pressure side). Fat blades have more tip than narrow blades, so they're less efficient.
      $endgroup$
      – Robin Bennett
      Aug 1 at 11:24






    • 3




      $begingroup$
      Submarines, which have no "sticking out of the water" problem, can have incredibly large props.
      $endgroup$
      – Carl Witthoft
      Aug 1 at 12:41










    • $begingroup$
      @CarlWitthoft submarines have to work to a degree on the surface. And a screw that sticks out of the water is one someone can take a picture of... if the boat comes out of the water in drydock the screw gets shrouded. It might be more accurate to say that submarines have very cramped hulls.
      $endgroup$
      – Chris Stratton
      Aug 1 at 14:57







    • 4




      $begingroup$
      @Luaan Some household fan blades actually do spin at similar RPMs to light aircraft propellers. They just tend to be rather small fans and usually have a shroud around them to prevent any danger. And they're also not nearly as long, so the blade tips aren't moving nearly as fast for the same RPMs. Also, not nearly as much mass per blade. Computer cooling fans are a common example of this. It's not unusual for them to be in the 1,500 - 2,000 RPM range, which is around the same as the PA-28s I've flown.
      $endgroup$
      – reirab
      Aug 1 at 16:48








    6




    6




    $begingroup$
    Fan blades spinning as fast as propellers would also be a safety hazard, and also probably have extra trouble with vibrations. It's just a bad idea all around :D
    $endgroup$
    – Luaan
    Aug 1 at 7:10




    $begingroup$
    Fan blades spinning as fast as propellers would also be a safety hazard, and also probably have extra trouble with vibrations. It's just a bad idea all around :D
    $endgroup$
    – Luaan
    Aug 1 at 7:10




    1




    1




    $begingroup$
    The key reason for this is that efficiency loss mainly occurs at the tip of a blade (where high pressure air can leak around to the low pressure side). Fat blades have more tip than narrow blades, so they're less efficient.
    $endgroup$
    – Robin Bennett
    Aug 1 at 11:24




    $begingroup$
    The key reason for this is that efficiency loss mainly occurs at the tip of a blade (where high pressure air can leak around to the low pressure side). Fat blades have more tip than narrow blades, so they're less efficient.
    $endgroup$
    – Robin Bennett
    Aug 1 at 11:24




    3




    3




    $begingroup$
    Submarines, which have no "sticking out of the water" problem, can have incredibly large props.
    $endgroup$
    – Carl Witthoft
    Aug 1 at 12:41




    $begingroup$
    Submarines, which have no "sticking out of the water" problem, can have incredibly large props.
    $endgroup$
    – Carl Witthoft
    Aug 1 at 12:41












    $begingroup$
    @CarlWitthoft submarines have to work to a degree on the surface. And a screw that sticks out of the water is one someone can take a picture of... if the boat comes out of the water in drydock the screw gets shrouded. It might be more accurate to say that submarines have very cramped hulls.
    $endgroup$
    – Chris Stratton
    Aug 1 at 14:57





    $begingroup$
    @CarlWitthoft submarines have to work to a degree on the surface. And a screw that sticks out of the water is one someone can take a picture of... if the boat comes out of the water in drydock the screw gets shrouded. It might be more accurate to say that submarines have very cramped hulls.
    $endgroup$
    – Chris Stratton
    Aug 1 at 14:57





    4




    4




    $begingroup$
    @Luaan Some household fan blades actually do spin at similar RPMs to light aircraft propellers. They just tend to be rather small fans and usually have a shroud around them to prevent any danger. And they're also not nearly as long, so the blade tips aren't moving nearly as fast for the same RPMs. Also, not nearly as much mass per blade. Computer cooling fans are a common example of this. It's not unusual for them to be in the 1,500 - 2,000 RPM range, which is around the same as the PA-28s I've flown.
    $endgroup$
    – reirab
    Aug 1 at 16:48





    $begingroup$
    @Luaan Some household fan blades actually do spin at similar RPMs to light aircraft propellers. They just tend to be rather small fans and usually have a shroud around them to prevent any danger. And they're also not nearly as long, so the blade tips aren't moving nearly as fast for the same RPMs. Also, not nearly as much mass per blade. Computer cooling fans are a common example of this. It's not unusual for them to be in the 1,500 - 2,000 RPM range, which is around the same as the PA-28s I've flown.
    $endgroup$
    – reirab
    Aug 1 at 16:48














    22
















    $begingroup$

    From our local warehouse



    Some household fans are shaped like airplane propellers, the ones that need to move a lot of air at the highest speed. For a given motor, they have the highest efficiency, but the tip effect makes them noisy. They are best suited for industrial applications.



    For inside the family home there are other considerations:



    • Silence. Best if we don’t hear the fan run at all.

    • Low air speed. We only want to feel the cooling effect of moving air, not have our hair blown out of shape.

    • Purchase cost. This is the one we directly see. The usage costs are hidden in the monthly electricity bill.

    So for that purpose, a slow moving tip with many broad blades is best.






    share|improve this answer












    $endgroup$














    • $begingroup$
      Let's try helicopter vs ceiling fan: helicopter - symmetrical airfoil. Ceiling fan - flat. Note long thin fan blades will help reduce interblade turbulence issues for a high speed industrial fan, but optimal airmoving shape that produces the LOWEST "lift" and LEAST DRAG would likely be a thin plate.
      $endgroup$
      – Robert DiGiovanni
      Jul 31 at 22:12







    • 6




      $begingroup$
      @RobertDiGiovanni ceiling fans tend towards flat blades because of their lower production cost and because they are often intended to operate in reverse with some efficiency, not because an airfoil would be bad. I have owned several standing fans with airfoil shaped blades, as well.
      $endgroup$
      – AEhere supports Monica
      Jul 31 at 22:24






    • 2




      $begingroup$
      "usage costs are hidden in the monthly electricity bill." Maybe about 0.5-2$ USD per month?
      $endgroup$
      – chux - Reinstate Monica
      Aug 1 at 7:06






    • 1




      $begingroup$
      @chux $2 per month adds up to the purchase price of a cheap desktop fan in a year. Of course you can pay 10 times as much for a fan that does exactly the same thing, if you want to :)
      $endgroup$
      – alephzero
      Aug 1 at 11:23















    22
















    $begingroup$

    From our local warehouse



    Some household fans are shaped like airplane propellers, the ones that need to move a lot of air at the highest speed. For a given motor, they have the highest efficiency, but the tip effect makes them noisy. They are best suited for industrial applications.



    For inside the family home there are other considerations:



    • Silence. Best if we don’t hear the fan run at all.

    • Low air speed. We only want to feel the cooling effect of moving air, not have our hair blown out of shape.

    • Purchase cost. This is the one we directly see. The usage costs are hidden in the monthly electricity bill.

    So for that purpose, a slow moving tip with many broad blades is best.






    share|improve this answer












    $endgroup$














    • $begingroup$
      Let's try helicopter vs ceiling fan: helicopter - symmetrical airfoil. Ceiling fan - flat. Note long thin fan blades will help reduce interblade turbulence issues for a high speed industrial fan, but optimal airmoving shape that produces the LOWEST "lift" and LEAST DRAG would likely be a thin plate.
      $endgroup$
      – Robert DiGiovanni
      Jul 31 at 22:12







    • 6




      $begingroup$
      @RobertDiGiovanni ceiling fans tend towards flat blades because of their lower production cost and because they are often intended to operate in reverse with some efficiency, not because an airfoil would be bad. I have owned several standing fans with airfoil shaped blades, as well.
      $endgroup$
      – AEhere supports Monica
      Jul 31 at 22:24






    • 2




      $begingroup$
      "usage costs are hidden in the monthly electricity bill." Maybe about 0.5-2$ USD per month?
      $endgroup$
      – chux - Reinstate Monica
      Aug 1 at 7:06






    • 1




      $begingroup$
      @chux $2 per month adds up to the purchase price of a cheap desktop fan in a year. Of course you can pay 10 times as much for a fan that does exactly the same thing, if you want to :)
      $endgroup$
      – alephzero
      Aug 1 at 11:23













    22














    22










    22







    $begingroup$

    From our local warehouse



    Some household fans are shaped like airplane propellers, the ones that need to move a lot of air at the highest speed. For a given motor, they have the highest efficiency, but the tip effect makes them noisy. They are best suited for industrial applications.



    For inside the family home there are other considerations:



    • Silence. Best if we don’t hear the fan run at all.

    • Low air speed. We only want to feel the cooling effect of moving air, not have our hair blown out of shape.

    • Purchase cost. This is the one we directly see. The usage costs are hidden in the monthly electricity bill.

    So for that purpose, a slow moving tip with many broad blades is best.






    share|improve this answer












    $endgroup$



    From our local warehouse



    Some household fans are shaped like airplane propellers, the ones that need to move a lot of air at the highest speed. For a given motor, they have the highest efficiency, but the tip effect makes them noisy. They are best suited for industrial applications.



    For inside the family home there are other considerations:



    • Silence. Best if we don’t hear the fan run at all.

    • Low air speed. We only want to feel the cooling effect of moving air, not have our hair blown out of shape.

    • Purchase cost. This is the one we directly see. The usage costs are hidden in the monthly electricity bill.

    So for that purpose, a slow moving tip with many broad blades is best.







    share|improve this answer















    share|improve this answer




    share|improve this answer








    edited Aug 1 at 21:18









    Peter Mortensen

    3352 silver badges7 bronze badges




    3352 silver badges7 bronze badges










    answered Jul 31 at 21:44









    KoyovisKoyovis

    39k9 gold badges106 silver badges200 bronze badges




    39k9 gold badges106 silver badges200 bronze badges














    • $begingroup$
      Let's try helicopter vs ceiling fan: helicopter - symmetrical airfoil. Ceiling fan - flat. Note long thin fan blades will help reduce interblade turbulence issues for a high speed industrial fan, but optimal airmoving shape that produces the LOWEST "lift" and LEAST DRAG would likely be a thin plate.
      $endgroup$
      – Robert DiGiovanni
      Jul 31 at 22:12







    • 6




      $begingroup$
      @RobertDiGiovanni ceiling fans tend towards flat blades because of their lower production cost and because they are often intended to operate in reverse with some efficiency, not because an airfoil would be bad. I have owned several standing fans with airfoil shaped blades, as well.
      $endgroup$
      – AEhere supports Monica
      Jul 31 at 22:24






    • 2




      $begingroup$
      "usage costs are hidden in the monthly electricity bill." Maybe about 0.5-2$ USD per month?
      $endgroup$
      – chux - Reinstate Monica
      Aug 1 at 7:06






    • 1




      $begingroup$
      @chux $2 per month adds up to the purchase price of a cheap desktop fan in a year. Of course you can pay 10 times as much for a fan that does exactly the same thing, if you want to :)
      $endgroup$
      – alephzero
      Aug 1 at 11:23
















    • $begingroup$
      Let's try helicopter vs ceiling fan: helicopter - symmetrical airfoil. Ceiling fan - flat. Note long thin fan blades will help reduce interblade turbulence issues for a high speed industrial fan, but optimal airmoving shape that produces the LOWEST "lift" and LEAST DRAG would likely be a thin plate.
      $endgroup$
      – Robert DiGiovanni
      Jul 31 at 22:12







    • 6




      $begingroup$
      @RobertDiGiovanni ceiling fans tend towards flat blades because of their lower production cost and because they are often intended to operate in reverse with some efficiency, not because an airfoil would be bad. I have owned several standing fans with airfoil shaped blades, as well.
      $endgroup$
      – AEhere supports Monica
      Jul 31 at 22:24






    • 2




      $begingroup$
      "usage costs are hidden in the monthly electricity bill." Maybe about 0.5-2$ USD per month?
      $endgroup$
      – chux - Reinstate Monica
      Aug 1 at 7:06






    • 1




      $begingroup$
      @chux $2 per month adds up to the purchase price of a cheap desktop fan in a year. Of course you can pay 10 times as much for a fan that does exactly the same thing, if you want to :)
      $endgroup$
      – alephzero
      Aug 1 at 11:23















    $begingroup$
    Let's try helicopter vs ceiling fan: helicopter - symmetrical airfoil. Ceiling fan - flat. Note long thin fan blades will help reduce interblade turbulence issues for a high speed industrial fan, but optimal airmoving shape that produces the LOWEST "lift" and LEAST DRAG would likely be a thin plate.
    $endgroup$
    – Robert DiGiovanni
    Jul 31 at 22:12





    $begingroup$
    Let's try helicopter vs ceiling fan: helicopter - symmetrical airfoil. Ceiling fan - flat. Note long thin fan blades will help reduce interblade turbulence issues for a high speed industrial fan, but optimal airmoving shape that produces the LOWEST "lift" and LEAST DRAG would likely be a thin plate.
    $endgroup$
    – Robert DiGiovanni
    Jul 31 at 22:12





    6




    6




    $begingroup$
    @RobertDiGiovanni ceiling fans tend towards flat blades because of their lower production cost and because they are often intended to operate in reverse with some efficiency, not because an airfoil would be bad. I have owned several standing fans with airfoil shaped blades, as well.
    $endgroup$
    – AEhere supports Monica
    Jul 31 at 22:24




    $begingroup$
    @RobertDiGiovanni ceiling fans tend towards flat blades because of their lower production cost and because they are often intended to operate in reverse with some efficiency, not because an airfoil would be bad. I have owned several standing fans with airfoil shaped blades, as well.
    $endgroup$
    – AEhere supports Monica
    Jul 31 at 22:24




    2




    2




    $begingroup$
    "usage costs are hidden in the monthly electricity bill." Maybe about 0.5-2$ USD per month?
    $endgroup$
    – chux - Reinstate Monica
    Aug 1 at 7:06




    $begingroup$
    "usage costs are hidden in the monthly electricity bill." Maybe about 0.5-2$ USD per month?
    $endgroup$
    – chux - Reinstate Monica
    Aug 1 at 7:06




    1




    1




    $begingroup$
    @chux $2 per month adds up to the purchase price of a cheap desktop fan in a year. Of course you can pay 10 times as much for a fan that does exactly the same thing, if you want to :)
    $endgroup$
    – alephzero
    Aug 1 at 11:23




    $begingroup$
    @chux $2 per month adds up to the purchase price of a cheap desktop fan in a year. Of course you can pay 10 times as much for a fan that does exactly the same thing, if you want to :)
    $endgroup$
    – alephzero
    Aug 1 at 11:23











    8
















    $begingroup$

    I think it should be obvious that a fan blade is basically a wing travelling in a circle. So the question is what makes a wing a good wing?



    In theory and in practice (wind tunnel measurements or fuel consumption measurements of real planes) a long skinny wing is significantly more efficient than a short wide wing. The most efficient wing for a given airfoil is actually one with infinite wingspan. But since there are only a finite number of electrons in the universe it is not possible to build this perfect wing. Instead engineers try to make wings/propellers/fan blades as long and as skinny as practical given the limitations of materials.



    You will even see this effect in household fan blades: in situations where span is not limited, such as ceiling fans, the fan blades tend to be long and skinny instead of short and wide like ship propellers. So why short and wide? Space restriction.



    When the fan must fit on a tabletop, you cannot have 1 meter long blades. But as soon as this restriction is lifted you will see designers shift to long skinny blades like what you can find on industrial floor standing or wall mounted fans.



    Granted, there are floor standing fans with the exact same blades as tabletop fans but this has more to do with reusing parts and economies of scale than aerodynamics.



    Now, you can in theory also have an efficient short skinny fan on a tabletop fan: just look at drone/RC airplane propellers - they're often way shorter than the fan blades of tabletop fans. But you will need to move the blades really fast to move a given amount of air. This is very noisy (have you ever seen a drone fly?). So the second consideration is noise. To reduce noise you move the blades slowly. A slow blade doesn't move much air so you increase the chord to make them wide. This results in a very inefficient fan but efficiency is not your main concern: you are designing a non-moving machine sitting on a table drawing power from a socket in the wall. Noise reduction and size matters more.






    share|improve this answer










    $endgroup$










    • 4




      $begingroup$
      +1 As an example, I used to pass through the VLC airport in the past, and it has massive fans on the ceiling, with long and thin blades not unlike the wings on a glider, and with a similar airfoil too. The fact that building maintenance often had them rotating backwards can be explained via Hanlon's razor. EDIT: see this question aviation.stackexchange.com/questions/61833/…
      $endgroup$
      – AEhere supports Monica
      Aug 1 at 12:56
















    8
















    $begingroup$

    I think it should be obvious that a fan blade is basically a wing travelling in a circle. So the question is what makes a wing a good wing?



    In theory and in practice (wind tunnel measurements or fuel consumption measurements of real planes) a long skinny wing is significantly more efficient than a short wide wing. The most efficient wing for a given airfoil is actually one with infinite wingspan. But since there are only a finite number of electrons in the universe it is not possible to build this perfect wing. Instead engineers try to make wings/propellers/fan blades as long and as skinny as practical given the limitations of materials.



    You will even see this effect in household fan blades: in situations where span is not limited, such as ceiling fans, the fan blades tend to be long and skinny instead of short and wide like ship propellers. So why short and wide? Space restriction.



    When the fan must fit on a tabletop, you cannot have 1 meter long blades. But as soon as this restriction is lifted you will see designers shift to long skinny blades like what you can find on industrial floor standing or wall mounted fans.



    Granted, there are floor standing fans with the exact same blades as tabletop fans but this has more to do with reusing parts and economies of scale than aerodynamics.



    Now, you can in theory also have an efficient short skinny fan on a tabletop fan: just look at drone/RC airplane propellers - they're often way shorter than the fan blades of tabletop fans. But you will need to move the blades really fast to move a given amount of air. This is very noisy (have you ever seen a drone fly?). So the second consideration is noise. To reduce noise you move the blades slowly. A slow blade doesn't move much air so you increase the chord to make them wide. This results in a very inefficient fan but efficiency is not your main concern: you are designing a non-moving machine sitting on a table drawing power from a socket in the wall. Noise reduction and size matters more.






    share|improve this answer










    $endgroup$










    • 4




      $begingroup$
      +1 As an example, I used to pass through the VLC airport in the past, and it has massive fans on the ceiling, with long and thin blades not unlike the wings on a glider, and with a similar airfoil too. The fact that building maintenance often had them rotating backwards can be explained via Hanlon's razor. EDIT: see this question aviation.stackexchange.com/questions/61833/…
      $endgroup$
      – AEhere supports Monica
      Aug 1 at 12:56














    8














    8










    8







    $begingroup$

    I think it should be obvious that a fan blade is basically a wing travelling in a circle. So the question is what makes a wing a good wing?



    In theory and in practice (wind tunnel measurements or fuel consumption measurements of real planes) a long skinny wing is significantly more efficient than a short wide wing. The most efficient wing for a given airfoil is actually one with infinite wingspan. But since there are only a finite number of electrons in the universe it is not possible to build this perfect wing. Instead engineers try to make wings/propellers/fan blades as long and as skinny as practical given the limitations of materials.



    You will even see this effect in household fan blades: in situations where span is not limited, such as ceiling fans, the fan blades tend to be long and skinny instead of short and wide like ship propellers. So why short and wide? Space restriction.



    When the fan must fit on a tabletop, you cannot have 1 meter long blades. But as soon as this restriction is lifted you will see designers shift to long skinny blades like what you can find on industrial floor standing or wall mounted fans.



    Granted, there are floor standing fans with the exact same blades as tabletop fans but this has more to do with reusing parts and economies of scale than aerodynamics.



    Now, you can in theory also have an efficient short skinny fan on a tabletop fan: just look at drone/RC airplane propellers - they're often way shorter than the fan blades of tabletop fans. But you will need to move the blades really fast to move a given amount of air. This is very noisy (have you ever seen a drone fly?). So the second consideration is noise. To reduce noise you move the blades slowly. A slow blade doesn't move much air so you increase the chord to make them wide. This results in a very inefficient fan but efficiency is not your main concern: you are designing a non-moving machine sitting on a table drawing power from a socket in the wall. Noise reduction and size matters more.






    share|improve this answer










    $endgroup$



    I think it should be obvious that a fan blade is basically a wing travelling in a circle. So the question is what makes a wing a good wing?



    In theory and in practice (wind tunnel measurements or fuel consumption measurements of real planes) a long skinny wing is significantly more efficient than a short wide wing. The most efficient wing for a given airfoil is actually one with infinite wingspan. But since there are only a finite number of electrons in the universe it is not possible to build this perfect wing. Instead engineers try to make wings/propellers/fan blades as long and as skinny as practical given the limitations of materials.



    You will even see this effect in household fan blades: in situations where span is not limited, such as ceiling fans, the fan blades tend to be long and skinny instead of short and wide like ship propellers. So why short and wide? Space restriction.



    When the fan must fit on a tabletop, you cannot have 1 meter long blades. But as soon as this restriction is lifted you will see designers shift to long skinny blades like what you can find on industrial floor standing or wall mounted fans.



    Granted, there are floor standing fans with the exact same blades as tabletop fans but this has more to do with reusing parts and economies of scale than aerodynamics.



    Now, you can in theory also have an efficient short skinny fan on a tabletop fan: just look at drone/RC airplane propellers - they're often way shorter than the fan blades of tabletop fans. But you will need to move the blades really fast to move a given amount of air. This is very noisy (have you ever seen a drone fly?). So the second consideration is noise. To reduce noise you move the blades slowly. A slow blade doesn't move much air so you increase the chord to make them wide. This results in a very inefficient fan but efficiency is not your main concern: you are designing a non-moving machine sitting on a table drawing power from a socket in the wall. Noise reduction and size matters more.







    share|improve this answer













    share|improve this answer




    share|improve this answer










    answered Aug 1 at 11:20









    slebetmanslebetman

    1,4517 silver badges12 bronze badges




    1,4517 silver badges12 bronze badges










    • 4




      $begingroup$
      +1 As an example, I used to pass through the VLC airport in the past, and it has massive fans on the ceiling, with long and thin blades not unlike the wings on a glider, and with a similar airfoil too. The fact that building maintenance often had them rotating backwards can be explained via Hanlon's razor. EDIT: see this question aviation.stackexchange.com/questions/61833/…
      $endgroup$
      – AEhere supports Monica
      Aug 1 at 12:56













    • 4




      $begingroup$
      +1 As an example, I used to pass through the VLC airport in the past, and it has massive fans on the ceiling, with long and thin blades not unlike the wings on a glider, and with a similar airfoil too. The fact that building maintenance often had them rotating backwards can be explained via Hanlon's razor. EDIT: see this question aviation.stackexchange.com/questions/61833/…
      $endgroup$
      – AEhere supports Monica
      Aug 1 at 12:56








    4




    4




    $begingroup$
    +1 As an example, I used to pass through the VLC airport in the past, and it has massive fans on the ceiling, with long and thin blades not unlike the wings on a glider, and with a similar airfoil too. The fact that building maintenance often had them rotating backwards can be explained via Hanlon's razor. EDIT: see this question aviation.stackexchange.com/questions/61833/…
    $endgroup$
    – AEhere supports Monica
    Aug 1 at 12:56





    $begingroup$
    +1 As an example, I used to pass through the VLC airport in the past, and it has massive fans on the ceiling, with long and thin blades not unlike the wings on a glider, and with a similar airfoil too. The fact that building maintenance often had them rotating backwards can be explained via Hanlon's razor. EDIT: see this question aviation.stackexchange.com/questions/61833/…
    $endgroup$
    – AEhere supports Monica
    Aug 1 at 12:56












    2
















    $begingroup$

    Learning to flying and modelling a fan blade, both spin in the air but the propeller tries to move the plane, and the fan tries to move the air.



    How does this factor into design? Propellers stand above and apart from fans in that they generate lift not only by deflecting air (bottom lift), but also from their motion through the air (top lift). This is best explained by viewing the lift curve vs AOA of an airfoil from 0 to 45 degrees. Lift will increase up to stall, then decrease, then increase again up to 45 degrees. This means you have to move LESS air to get the same lift with an airfoil up to stall.



    The fan really is opposite, unless you want it to pull it self around the room. The fan blade is designed to move air, period. Sensible design would make it compact, and thin flat blades with a wider chord would be fine, as one just looking for a cooling breeze from their product. Keeping the original style of fan on the "antique" may make it more valuable.






    share|improve this answer












    $endgroup$










    • 2




      $begingroup$
      You have a good point, but it doesn't need to involve the dubious concepts of 'top' and 'bottom' lift. Simply put, propellers are optimised to create lift (thrust) while disturbing the air as little as possible. This favours a large diameter in the first place, to move more air with the lowest possible speed. A fan is almost the opposite: it needs a certain diameter and move the air at a decent speed. (A cooling fan, that is; requirements may be different for a ventilation fan).
      $endgroup$
      – Zeus
      Aug 1 at 1:24










    • $begingroup$
      "A cooling fan, that is; requirements may be different for a ventilation fan" which may be different from a jet fan! Look at the shape of those airfoils!
      $endgroup$
      – Robert DiGiovanni
      Aug 1 at 6:10






    • 1




      $begingroup$
      @Zeus Tanner Swett published an airfoil Clift vs AOA graph that shows the "top lift" contribution at (lower drag) lower AOA as a lift maxima, which drops to a minima at around 30 degrees AOA, before rising to a maxima (with loads of drag) at 45 degrees. This is the Rosetta Stone for many applications. However, your point that the moving wing "disturbs as little air as possible" is an excellent way of describing it (hence less drag for a given lift!). Thanks.
      $endgroup$
      – Robert DiGiovanni
      Aug 1 at 6:19










    • $begingroup$
      @RobertDiGiovanni Friendly nitpick: consider using the singular form of minimum and maximum when appropriate.
      $endgroup$
      – AEhere supports Monica
      Aug 1 at 7:37















    2
















    $begingroup$

    Learning to flying and modelling a fan blade, both spin in the air but the propeller tries to move the plane, and the fan tries to move the air.



    How does this factor into design? Propellers stand above and apart from fans in that they generate lift not only by deflecting air (bottom lift), but also from their motion through the air (top lift). This is best explained by viewing the lift curve vs AOA of an airfoil from 0 to 45 degrees. Lift will increase up to stall, then decrease, then increase again up to 45 degrees. This means you have to move LESS air to get the same lift with an airfoil up to stall.



    The fan really is opposite, unless you want it to pull it self around the room. The fan blade is designed to move air, period. Sensible design would make it compact, and thin flat blades with a wider chord would be fine, as one just looking for a cooling breeze from their product. Keeping the original style of fan on the "antique" may make it more valuable.






    share|improve this answer












    $endgroup$










    • 2




      $begingroup$
      You have a good point, but it doesn't need to involve the dubious concepts of 'top' and 'bottom' lift. Simply put, propellers are optimised to create lift (thrust) while disturbing the air as little as possible. This favours a large diameter in the first place, to move more air with the lowest possible speed. A fan is almost the opposite: it needs a certain diameter and move the air at a decent speed. (A cooling fan, that is; requirements may be different for a ventilation fan).
      $endgroup$
      – Zeus
      Aug 1 at 1:24










    • $begingroup$
      "A cooling fan, that is; requirements may be different for a ventilation fan" which may be different from a jet fan! Look at the shape of those airfoils!
      $endgroup$
      – Robert DiGiovanni
      Aug 1 at 6:10






    • 1




      $begingroup$
      @Zeus Tanner Swett published an airfoil Clift vs AOA graph that shows the "top lift" contribution at (lower drag) lower AOA as a lift maxima, which drops to a minima at around 30 degrees AOA, before rising to a maxima (with loads of drag) at 45 degrees. This is the Rosetta Stone for many applications. However, your point that the moving wing "disturbs as little air as possible" is an excellent way of describing it (hence less drag for a given lift!). Thanks.
      $endgroup$
      – Robert DiGiovanni
      Aug 1 at 6:19










    • $begingroup$
      @RobertDiGiovanni Friendly nitpick: consider using the singular form of minimum and maximum when appropriate.
      $endgroup$
      – AEhere supports Monica
      Aug 1 at 7:37













    2














    2










    2







    $begingroup$

    Learning to flying and modelling a fan blade, both spin in the air but the propeller tries to move the plane, and the fan tries to move the air.



    How does this factor into design? Propellers stand above and apart from fans in that they generate lift not only by deflecting air (bottom lift), but also from their motion through the air (top lift). This is best explained by viewing the lift curve vs AOA of an airfoil from 0 to 45 degrees. Lift will increase up to stall, then decrease, then increase again up to 45 degrees. This means you have to move LESS air to get the same lift with an airfoil up to stall.



    The fan really is opposite, unless you want it to pull it self around the room. The fan blade is designed to move air, period. Sensible design would make it compact, and thin flat blades with a wider chord would be fine, as one just looking for a cooling breeze from their product. Keeping the original style of fan on the "antique" may make it more valuable.






    share|improve this answer












    $endgroup$



    Learning to flying and modelling a fan blade, both spin in the air but the propeller tries to move the plane, and the fan tries to move the air.



    How does this factor into design? Propellers stand above and apart from fans in that they generate lift not only by deflecting air (bottom lift), but also from their motion through the air (top lift). This is best explained by viewing the lift curve vs AOA of an airfoil from 0 to 45 degrees. Lift will increase up to stall, then decrease, then increase again up to 45 degrees. This means you have to move LESS air to get the same lift with an airfoil up to stall.



    The fan really is opposite, unless you want it to pull it self around the room. The fan blade is designed to move air, period. Sensible design would make it compact, and thin flat blades with a wider chord would be fine, as one just looking for a cooling breeze from their product. Keeping the original style of fan on the "antique" may make it more valuable.







    share|improve this answer















    share|improve this answer




    share|improve this answer








    edited Jul 31 at 19:41

























    answered Jul 31 at 19:32









    Robert DiGiovanniRobert DiGiovanni

    5,5961 gold badge6 silver badges29 bronze badges




    5,5961 gold badge6 silver badges29 bronze badges










    • 2




      $begingroup$
      You have a good point, but it doesn't need to involve the dubious concepts of 'top' and 'bottom' lift. Simply put, propellers are optimised to create lift (thrust) while disturbing the air as little as possible. This favours a large diameter in the first place, to move more air with the lowest possible speed. A fan is almost the opposite: it needs a certain diameter and move the air at a decent speed. (A cooling fan, that is; requirements may be different for a ventilation fan).
      $endgroup$
      – Zeus
      Aug 1 at 1:24










    • $begingroup$
      "A cooling fan, that is; requirements may be different for a ventilation fan" which may be different from a jet fan! Look at the shape of those airfoils!
      $endgroup$
      – Robert DiGiovanni
      Aug 1 at 6:10






    • 1




      $begingroup$
      @Zeus Tanner Swett published an airfoil Clift vs AOA graph that shows the "top lift" contribution at (lower drag) lower AOA as a lift maxima, which drops to a minima at around 30 degrees AOA, before rising to a maxima (with loads of drag) at 45 degrees. This is the Rosetta Stone for many applications. However, your point that the moving wing "disturbs as little air as possible" is an excellent way of describing it (hence less drag for a given lift!). Thanks.
      $endgroup$
      – Robert DiGiovanni
      Aug 1 at 6:19










    • $begingroup$
      @RobertDiGiovanni Friendly nitpick: consider using the singular form of minimum and maximum when appropriate.
      $endgroup$
      – AEhere supports Monica
      Aug 1 at 7:37












    • 2




      $begingroup$
      You have a good point, but it doesn't need to involve the dubious concepts of 'top' and 'bottom' lift. Simply put, propellers are optimised to create lift (thrust) while disturbing the air as little as possible. This favours a large diameter in the first place, to move more air with the lowest possible speed. A fan is almost the opposite: it needs a certain diameter and move the air at a decent speed. (A cooling fan, that is; requirements may be different for a ventilation fan).
      $endgroup$
      – Zeus
      Aug 1 at 1:24










    • $begingroup$
      "A cooling fan, that is; requirements may be different for a ventilation fan" which may be different from a jet fan! Look at the shape of those airfoils!
      $endgroup$
      – Robert DiGiovanni
      Aug 1 at 6:10






    • 1




      $begingroup$
      @Zeus Tanner Swett published an airfoil Clift vs AOA graph that shows the "top lift" contribution at (lower drag) lower AOA as a lift maxima, which drops to a minima at around 30 degrees AOA, before rising to a maxima (with loads of drag) at 45 degrees. This is the Rosetta Stone for many applications. However, your point that the moving wing "disturbs as little air as possible" is an excellent way of describing it (hence less drag for a given lift!). Thanks.
      $endgroup$
      – Robert DiGiovanni
      Aug 1 at 6:19










    • $begingroup$
      @RobertDiGiovanni Friendly nitpick: consider using the singular form of minimum and maximum when appropriate.
      $endgroup$
      – AEhere supports Monica
      Aug 1 at 7:37







    2




    2




    $begingroup$
    You have a good point, but it doesn't need to involve the dubious concepts of 'top' and 'bottom' lift. Simply put, propellers are optimised to create lift (thrust) while disturbing the air as little as possible. This favours a large diameter in the first place, to move more air with the lowest possible speed. A fan is almost the opposite: it needs a certain diameter and move the air at a decent speed. (A cooling fan, that is; requirements may be different for a ventilation fan).
    $endgroup$
    – Zeus
    Aug 1 at 1:24




    $begingroup$
    You have a good point, but it doesn't need to involve the dubious concepts of 'top' and 'bottom' lift. Simply put, propellers are optimised to create lift (thrust) while disturbing the air as little as possible. This favours a large diameter in the first place, to move more air with the lowest possible speed. A fan is almost the opposite: it needs a certain diameter and move the air at a decent speed. (A cooling fan, that is; requirements may be different for a ventilation fan).
    $endgroup$
    – Zeus
    Aug 1 at 1:24












    $begingroup$
    "A cooling fan, that is; requirements may be different for a ventilation fan" which may be different from a jet fan! Look at the shape of those airfoils!
    $endgroup$
    – Robert DiGiovanni
    Aug 1 at 6:10




    $begingroup$
    "A cooling fan, that is; requirements may be different for a ventilation fan" which may be different from a jet fan! Look at the shape of those airfoils!
    $endgroup$
    – Robert DiGiovanni
    Aug 1 at 6:10




    1




    1




    $begingroup$
    @Zeus Tanner Swett published an airfoil Clift vs AOA graph that shows the "top lift" contribution at (lower drag) lower AOA as a lift maxima, which drops to a minima at around 30 degrees AOA, before rising to a maxima (with loads of drag) at 45 degrees. This is the Rosetta Stone for many applications. However, your point that the moving wing "disturbs as little air as possible" is an excellent way of describing it (hence less drag for a given lift!). Thanks.
    $endgroup$
    – Robert DiGiovanni
    Aug 1 at 6:19




    $begingroup$
    @Zeus Tanner Swett published an airfoil Clift vs AOA graph that shows the "top lift" contribution at (lower drag) lower AOA as a lift maxima, which drops to a minima at around 30 degrees AOA, before rising to a maxima (with loads of drag) at 45 degrees. This is the Rosetta Stone for many applications. However, your point that the moving wing "disturbs as little air as possible" is an excellent way of describing it (hence less drag for a given lift!). Thanks.
    $endgroup$
    – Robert DiGiovanni
    Aug 1 at 6:19












    $begingroup$
    @RobertDiGiovanni Friendly nitpick: consider using the singular form of minimum and maximum when appropriate.
    $endgroup$
    – AEhere supports Monica
    Aug 1 at 7:37




    $begingroup$
    @RobertDiGiovanni Friendly nitpick: consider using the singular form of minimum and maximum when appropriate.
    $endgroup$
    – AEhere supports Monica
    Aug 1 at 7:37











    2
















    $begingroup$

    Price is the main reason. Ceiling fans just stir the air around and flat paddles are the cheapest way to go.



    More expensive fan blades can have some aerofoil shape and even winglets at the tips but this is mainly for show since these blades don't have twist.






    share|improve this answer










    $endgroup$














    • $begingroup$
      But wouldn't a more wing-shaped, narrower blade use less material and be less expensive?
      $endgroup$
      – schadjo
      Aug 1 at 15:08
















    2
















    $begingroup$

    Price is the main reason. Ceiling fans just stir the air around and flat paddles are the cheapest way to go.



    More expensive fan blades can have some aerofoil shape and even winglets at the tips but this is mainly for show since these blades don't have twist.






    share|improve this answer










    $endgroup$














    • $begingroup$
      But wouldn't a more wing-shaped, narrower blade use less material and be less expensive?
      $endgroup$
      – schadjo
      Aug 1 at 15:08














    2














    2










    2







    $begingroup$

    Price is the main reason. Ceiling fans just stir the air around and flat paddles are the cheapest way to go.



    More expensive fan blades can have some aerofoil shape and even winglets at the tips but this is mainly for show since these blades don't have twist.






    share|improve this answer










    $endgroup$



    Price is the main reason. Ceiling fans just stir the air around and flat paddles are the cheapest way to go.



    More expensive fan blades can have some aerofoil shape and even winglets at the tips but this is mainly for show since these blades don't have twist.







    share|improve this answer













    share|improve this answer




    share|improve this answer










    answered Aug 1 at 3:09









    Dermot McDermotDermot McDermot

    211 bronze badge




    211 bronze badge














    • $begingroup$
      But wouldn't a more wing-shaped, narrower blade use less material and be less expensive?
      $endgroup$
      – schadjo
      Aug 1 at 15:08

















    • $begingroup$
      But wouldn't a more wing-shaped, narrower blade use less material and be less expensive?
      $endgroup$
      – schadjo
      Aug 1 at 15:08
















    $begingroup$
    But wouldn't a more wing-shaped, narrower blade use less material and be less expensive?
    $endgroup$
    – schadjo
    Aug 1 at 15:08





    $begingroup$
    But wouldn't a more wing-shaped, narrower blade use less material and be less expensive?
    $endgroup$
    – schadjo
    Aug 1 at 15:08












    1
















    $begingroup$

    Most of the other answers are already correct. Just one more aspect: Blade chord length.



    With something that spins as fast as an airplane propeller, the blades need to be as light as possible, to minimize centrifugal loads. So it's better to give them a short profile chord length and high area loading (how much force they produce per surface area of blade), which means small chord length and turning the flow comparatively sharply, which requires a large pressure difference between the pressure and suction side of the fan blades. For a desk fan, any old plastic will be strong enough, so those blades can work with a very low surface loading (long profile chord length) and turn the flow slowly, with a small pressure difference, which is quieter.



    The same applies for ship propellers: Since water is so dense, the structural loading on those blades is pretty high. If you tried to get the same thrust from a propeller with short chords (at equal radius), the blades would have trouble carrying the bending loads, and the sharper turning would also lead to cavitation [1] quicker.



    [1]: cavitation happens when the pressure on the suction side of an underwater propeller falls so low that water evaporates. This effectively separates the flow, which reduces suction, so the freshly-evaporated water condenses again, water rushes back to where the steam bubble was... That can break a propeller very quickly.






    share|improve this answer










    $endgroup$



















      1
















      $begingroup$

      Most of the other answers are already correct. Just one more aspect: Blade chord length.



      With something that spins as fast as an airplane propeller, the blades need to be as light as possible, to minimize centrifugal loads. So it's better to give them a short profile chord length and high area loading (how much force they produce per surface area of blade), which means small chord length and turning the flow comparatively sharply, which requires a large pressure difference between the pressure and suction side of the fan blades. For a desk fan, any old plastic will be strong enough, so those blades can work with a very low surface loading (long profile chord length) and turn the flow slowly, with a small pressure difference, which is quieter.



      The same applies for ship propellers: Since water is so dense, the structural loading on those blades is pretty high. If you tried to get the same thrust from a propeller with short chords (at equal radius), the blades would have trouble carrying the bending loads, and the sharper turning would also lead to cavitation [1] quicker.



      [1]: cavitation happens when the pressure on the suction side of an underwater propeller falls so low that water evaporates. This effectively separates the flow, which reduces suction, so the freshly-evaporated water condenses again, water rushes back to where the steam bubble was... That can break a propeller very quickly.






      share|improve this answer










      $endgroup$

















        1














        1










        1







        $begingroup$

        Most of the other answers are already correct. Just one more aspect: Blade chord length.



        With something that spins as fast as an airplane propeller, the blades need to be as light as possible, to minimize centrifugal loads. So it's better to give them a short profile chord length and high area loading (how much force they produce per surface area of blade), which means small chord length and turning the flow comparatively sharply, which requires a large pressure difference between the pressure and suction side of the fan blades. For a desk fan, any old plastic will be strong enough, so those blades can work with a very low surface loading (long profile chord length) and turn the flow slowly, with a small pressure difference, which is quieter.



        The same applies for ship propellers: Since water is so dense, the structural loading on those blades is pretty high. If you tried to get the same thrust from a propeller with short chords (at equal radius), the blades would have trouble carrying the bending loads, and the sharper turning would also lead to cavitation [1] quicker.



        [1]: cavitation happens when the pressure on the suction side of an underwater propeller falls so low that water evaporates. This effectively separates the flow, which reduces suction, so the freshly-evaporated water condenses again, water rushes back to where the steam bubble was... That can break a propeller very quickly.






        share|improve this answer










        $endgroup$



        Most of the other answers are already correct. Just one more aspect: Blade chord length.



        With something that spins as fast as an airplane propeller, the blades need to be as light as possible, to minimize centrifugal loads. So it's better to give them a short profile chord length and high area loading (how much force they produce per surface area of blade), which means small chord length and turning the flow comparatively sharply, which requires a large pressure difference between the pressure and suction side of the fan blades. For a desk fan, any old plastic will be strong enough, so those blades can work with a very low surface loading (long profile chord length) and turn the flow slowly, with a small pressure difference, which is quieter.



        The same applies for ship propellers: Since water is so dense, the structural loading on those blades is pretty high. If you tried to get the same thrust from a propeller with short chords (at equal radius), the blades would have trouble carrying the bending loads, and the sharper turning would also lead to cavitation [1] quicker.



        [1]: cavitation happens when the pressure on the suction side of an underwater propeller falls so low that water evaporates. This effectively separates the flow, which reduces suction, so the freshly-evaporated water condenses again, water rushes back to where the steam bubble was... That can break a propeller very quickly.







        share|improve this answer













        share|improve this answer




        share|improve this answer










        answered Aug 4 at 2:50









        ZakZak

        2887 bronze badges




        2887 bronze badges
























            1
















            $begingroup$

            Some rare designs do look more like a fan blade. The Antonov AN-70 is a good example of this, sporting huge fan blades on its D-27 propfan. The scimitar props do make for a rather efficient design, but they are extremely noisy which limits their usability around many commercial airports.



            enter image description here



            Image credit : By Tangopaso - Own work, CC BY-SA 3.0




            Some additional discussion can be found in this question :



            What are the advantages of more than 4 propeller blades?






            share|improve this answer










            $endgroup$














            • $begingroup$
              Those props are wild.
              $endgroup$
              – schadjo
              Aug 13 at 12:11















            1
















            $begingroup$

            Some rare designs do look more like a fan blade. The Antonov AN-70 is a good example of this, sporting huge fan blades on its D-27 propfan. The scimitar props do make for a rather efficient design, but they are extremely noisy which limits their usability around many commercial airports.



            enter image description here



            Image credit : By Tangopaso - Own work, CC BY-SA 3.0




            Some additional discussion can be found in this question :



            What are the advantages of more than 4 propeller blades?






            share|improve this answer










            $endgroup$














            • $begingroup$
              Those props are wild.
              $endgroup$
              – schadjo
              Aug 13 at 12:11













            1














            1










            1







            $begingroup$

            Some rare designs do look more like a fan blade. The Antonov AN-70 is a good example of this, sporting huge fan blades on its D-27 propfan. The scimitar props do make for a rather efficient design, but they are extremely noisy which limits their usability around many commercial airports.



            enter image description here



            Image credit : By Tangopaso - Own work, CC BY-SA 3.0




            Some additional discussion can be found in this question :



            What are the advantages of more than 4 propeller blades?






            share|improve this answer










            $endgroup$



            Some rare designs do look more like a fan blade. The Antonov AN-70 is a good example of this, sporting huge fan blades on its D-27 propfan. The scimitar props do make for a rather efficient design, but they are extremely noisy which limits their usability around many commercial airports.



            enter image description here



            Image credit : By Tangopaso - Own work, CC BY-SA 3.0




            Some additional discussion can be found in this question :



            What are the advantages of more than 4 propeller blades?







            share|improve this answer













            share|improve this answer




            share|improve this answer










            answered Aug 12 at 19:56









            J...J...

            1,3158 silver badges12 bronze badges




            1,3158 silver badges12 bronze badges














            • $begingroup$
              Those props are wild.
              $endgroup$
              – schadjo
              Aug 13 at 12:11
















            • $begingroup$
              Those props are wild.
              $endgroup$
              – schadjo
              Aug 13 at 12:11















            $begingroup$
            Those props are wild.
            $endgroup$
            – schadjo
            Aug 13 at 12:11




            $begingroup$
            Those props are wild.
            $endgroup$
            – schadjo
            Aug 13 at 12:11


















            draft saved

            draft discarded















































            Thanks for contributing an answer to Aviation Stack Exchange!


            • Please be sure to answer the question. Provide details and share your research!

            But avoid


            • Asking for help, clarification, or responding to other answers.

            • Making statements based on opinion; back them up with references or personal experience.

            Use MathJax to format equations. MathJax reference.


            To learn more, see our tips on writing great answers.




            draft saved


            draft discarded














            StackExchange.ready(
            function ()
            StackExchange.openid.initPostLogin('.new-post-login', 'https%3a%2f%2faviation.stackexchange.com%2fquestions%2f67177%2fwhy-are-prop-blades-not-shaped-like-household-fan-blades%23new-answer', 'question_page');

            );

            Post as a guest















            Required, but never shown





















































            Required, but never shown














            Required, but never shown












            Required, but never shown







            Required, but never shown

































            Required, but never shown














            Required, but never shown












            Required, but never shown







            Required, but never shown









            Popular posts from this blog

            Tamil (spriik) Luke uk diar | Nawigatjuun

            Align equal signs while including text over equalitiesAMS align: left aligned text/math plus multicolumn alignmentMultiple alignmentsAligning equations in multiple placesNumbering and aligning an equation with multiple columnsHow to align one equation with another multline equationUsing \ in environments inside the begintabularxNumber equations and preserving alignment of equal signsHow can I align equations to the left and to the right?Double equation alignment problem within align enviromentAligned within align: Why are they right-aligned?

            Where does the image of a data connector as a sharp metal spike originate from?Where does the concept of infected people turning into zombies only after death originate from?Where does the motif of a reanimated human head originate?Where did the notion that Dragons could speak originate?Where does the archetypal image of the 'Grey' alien come from?Where did the suffix '-Man' originate?Where does the notion of being injured or killed by an illusion originate?Where did the term “sophont” originate?Where does the trope of magic spells being driven by advanced technology originate from?Where did the term “the living impaired” originate?